Đến nội dung

DangHuyNgheAn nội dung

Có 63 mục bởi DangHuyNgheAn (Tìm giới hạn từ 18-04-2020)



Sắp theo                Sắp xếp  

#541358 ĐỀ THI KIỂM TRA ĐỘI TUYỂN KHTN LỚP 10 VÒNG 2 NĂM 2015

Đã gửi bởi DangHuyNgheAn on 19-01-2015 - 21:06 trong Thi HSG cấp Tỉnh, Thành phố. Olympic 30-4. Đề thi và kiểm tra đội tuyển các cấp.

 Lấy pt (1)-(2) ,(2)-(3),(3-1) $= > \left\{\begin{matrix} 2(x-y)(x^2+xy+y^2)=3x(z-y) & & \\ 2(y-z)(y^2+yz+z^2)=3y(x-z) & & \\ 2(z-x)(z^2+xz+x^2)=3z(y-x) & & \end{matrix}\right.$

Nhân theo vế 

 

 $= > 8(x-y)(y-z)(z-x)\prod (x^2+xy+y^2)=27xyz(x-y)(y-z)(z-x)$

 

Nếu trong 3 tích trên có ít nhất 1 cái = 0 thì x=y hoặ y=z hoặc z=x .Thay vào đê bài rồi giải pt là xong

 

Nếu không có tích nào = 0 thì $8(x^2+xy+y^2)(y^2+yz+z^2)(z^2+xz+x^2)=27xyz$

 

Đến đây Cosi ta CM đc VT>VP  nên vo ly

Cosi the nao ha ban???




#540204 $2(x^2+y^2+z^2+1)(xy^3+yz^3+zx^3+xyz)\leq (x^2+y^2+z^2+3xyz)^2$

Đã gửi bởi DangHuyNgheAn on 09-01-2015 - 23:29 trong Bất đẳng thức và cực trị

Cho $x,y,z\geq 0$ thoả mãn: $x+y+z=1$. CMR:$2(x^2+y^2+z^2+1)(xy^3+yz^3+zx^3+xyz)\leq (x^2+y^2+z^2+3xyz)^2$




#540203 ĐỀ THI CHỌN HSG LỚP 9 SÔNG LÔ

Đã gửi bởi DangHuyNgheAn on 09-01-2015 - 23:23 trong Tài liệu - Đề thi

$Cau2. He\Leftrightarrow \left\{\begin{matrix}6a^2+4b^2+17a-7b=0 & \\ & a^3-b^3+35=0 \end{matrix}\right. (voi a=x+y,b=x-y).He nay nghiem le.=>decothesai$




#540201 ĐỀ THI CHỌN HSG LỚP 9 SÔNG LÔ

Đã gửi bởi DangHuyNgheAn on 09-01-2015 - 23:18 trong Tài liệu - Đề thi

Hai nghiem nay dau thoa man




#539350 Đề Thi Chọn Đội Tuyển Dự Thi Học Sinh Giỏi Tỉnh Huyện Yên Thành

Đã gửi bởi DangHuyNgheAn on 03-01-2015 - 20:34 trong Tài liệu - Đề thi

cau nao vay




#539345 Đề Thi Chọn Đội Tuyển Dự Thi Học Sinh Giỏi Tỉnh Huyện Yên Thành

Đã gửi bởi DangHuyNgheAn on 03-01-2015 - 20:28 trong Tài liệu - Đề thi

chú em lam het chứ




#539338 Đề Thi Chọn Đội Tuyển Dự Thi Học Sinh Giỏi Tỉnh Huyện Yên Thành

Đã gửi bởi DangHuyNgheAn on 03-01-2015 - 20:13 trong Tài liệu - Đề thi

vong may day ban.




#539303 $a^{m} - b^{m}=(a-b)^{n}$

Đã gửi bởi DangHuyNgheAn on 03-01-2015 - 18:05 trong Số học

$bai nay co tren THTT$




#538457 Chọn đội tuyển QG Tp Hải Phòng 2014-2015

Đã gửi bởi DangHuyNgheAn on 20-12-2014 - 16:37 trong Thi HSG cấp Tỉnh, Thành phố. Olympic 30-4. Đề thi và kiểm tra đội tuyển các cấp.

Bài hình ngày 1 thì quen rồi (nó trong dự tuyển IMO 199x gì đó,trong đề chọn đội tuyển tỉnh các năm trước,trong tài liệu chuyên toán hình học 10,.......)

Gọi $E,F$ là tiếp điểm của BC với $(O');(O'')$

Gọi $D'$ là điểm chính giữa cung $BC$ không chứa A.Dùng phép vị tự hoặc Talet ta được $M,E,D';N,F,D'$ đều thẳng hàng

$P_{D';(O')}=D'B.D'M=D'B^{2}=D'C^{2}=D'E.D'N=P_{D';(O'')}$$\Rightarrow$ D' thuộc trục đẳng phương của $(O'),(O'')\Rightarrow A,I,D'$ thẳng hàng (đpcm)

b,$DB^{2}=DI^{2}=DC^{2}=DE.DM\Rightarrow \bigtriangleup DIC$ cân ở D $\Rightarrow \angle DIC=\angle DCI\Rightarrow \frac{A}{2}+\angle ICA=\angle ICB+\frac{A}{2}\Rightarrow \angle ACI=\angle BCI\Rightarrow$ đpcm

Câu 3:Thay n bởi n+1 ta được $\frac{(n+2)^{3}}{3}a_{n+1}=(1.2a_{1}+2.3.a_{2}+...+n(n+1)a)+(n+1)(n+2)a_{n+1}$

Kết hợp với gt $a_{n}.\frac{(n+1)^{3}}{3}=1.2.a_{1}+2.3.a_{2}+...+n(n+1)a_{n}$ ta được $\frac{(n+2)^{3}}{3}a_{n+1}=\frac{(n+1)^{3}}{3}a_{n}+(n+1)(n+2)a_{n+1}\Leftrightarrow a_{n+1}=\frac{(n+1)^{3}}{n^{3}+5n^{2}+11n+10}a_{n}\Rightarrow a_{n+1}-a_{n}=\frac{-2n^{2}-8n-9}{n^{3}+5n^{2}+11n+10}a_{n}< 0\Rightarrow (a_{n})$ giảm và bị chặn dưới bởi 0 nên dãy hội tụ

\\Câu b $lima_{n}=0$ mới đúng chứ nhỉ?Chắc mình sai.........cần cao thủ trợ giúp??\\

Câu PTH Thay x=0 vào ta được $f(0+2f(y))=f(0)+\frac{y}{2}$.Dễ thấy $f$ đơn ánh (và cả toàn ánh nhưng không cần dùng)

Thay x=y=0$\Rightarrow f(2f(0))=f(0)\Rightarrow f(0)=0$.Khi đó $f(2f(y))=\frac{y}{2}$.

Thay $y=f(2y)$ (á phải dùng toàn ánh ) vào PTH ban đầu ta được $f(x+y)=f(x)+f(y)$ kết hợp với $f(x)\geq 0$ với mọi $x\geq 0\Rightarrow f$ cộng tính $\Rightarrow f(x)=x.$

Câu bất ngày 2 (korea MO 2014)  Bằng kỹ thuật thay b=c;và chọn k max sao cho bất đẳng thức luôn đúng ta chọn được $b=\frac{1}{2}$

Thay $a=0;b=c=\frac{1}{2}\Rightarrow k\leq 4$

Với $k=4$ ta cần cm $\sum \frac{a}{1+9bc+4(b-c)^{2}}\geq \frac{1}{2}$

AD Cauchy Shwars ta có $VY=\sum \frac{a^{2}}{a+9abc+4a(b-c)^{2}}\geq \frac{(a+b+c)^{2}}{(a+b+c)+27abc-24abc+ab(a+b)+bc(b+c)+ac(a+c)}=\frac{1}{1+3abc+\sum 4ab(1-c)}=\frac{1}{1+4(ab+bc+ac)-9abc}\geq \frac{1}{2}\Leftrightarrow 9abc\geq 4(ab+bc+ac)-1$

Nhưng điều này luôn đúng do $abc\geq \prod (a+b-c)=\prod (1-2a)\Rightarrow 9abc\geq 4(ab+bc+ac)-1$

Fdfdsf




#538422 $(\frac{10^6+1}{10^6})^{10^6} < 3...

Đã gửi bởi DangHuyNgheAn on 20-12-2014 - 11:46 trong Bất đẳng thức và cực trị

TQ $ (1+1/n)^n<3$




#538417 $Tìm Max P=3(xy+yz+zx)-xyz$

Đã gửi bởi DangHuyNgheAn on 20-12-2014 - 11:35 trong Bất đẳng thức và cực trị

Ban noi cai gi vay???




#538360 $Tìm Max P=3(xy+yz+zx)-xyz$

Đã gửi bởi DangHuyNgheAn on 17-12-2014 - 17:23 trong Bất đẳng thức và cực trị

$Cho:x,y,z khongamthoaman:x^3+y^3+z^3=3.Tim MaxP=3(xy+yz+zx)-xyz$




#538334 Giải hệ phương trình $\left\{\begin{matrix...

Đã gửi bởi DangHuyNgheAn on 17-12-2014 - 11:40 trong Phương trình - hệ phương trình - bất phương trình

Cau 2 thi de roi




#538333 Giải hệ phương trình $\left\{\begin{matrix...

Đã gửi bởi DangHuyNgheAn on 17-12-2014 - 11:38 trong Phương trình - hệ phương trình - bất phương trình

$CAu 1.(1)\Leftrightarrow (2x+y)(xy+1)=6xy\Leftrightarrow (2x+y)^2(1+\frac{1}{xy})^2=36.Kethop (2)=>8(4x^2+4xy+y^2)=36(x^2+y^2)\Leftrightarrow x=7y hoacx=y....$




#537022 Putnam 2014

Đã gửi bởi DangHuyNgheAn on 10-12-2014 - 16:51 trong Thảo luận về các kì thi, các kì kiểm tra Toán sinh viên

Cho minh hỏi cai nay la ki thi cua nuoc nao vay?




#534573 CMR: Với mọi số nguyên dương $n$ thỏa mãn $2^ny+1|x^{2^n...

Đã gửi bởi DangHuyNgheAn on 24-11-2014 - 17:36 trong Số học

IMO shorlist 2012




#533938 Bộ sưu tập TC Toán học và Tuổi trẻ từ năm 1994 đến nay

Đã gửi bởi DangHuyNgheAn on 20-11-2014 - 18:09 trong Toán học & Tuổi trẻ

Thày post báo tháng 11 đi thầy.




#533918 1. $6^x+8^x=10^x$

Đã gửi bởi DangHuyNgheAn on 20-11-2014 - 16:15 trong Số học

cách làm chưa dúng rồi.




#533916 $P=\sum \frac{a}{a^2+3}$

Đã gửi bởi DangHuyNgheAn on 20-11-2014 - 16:07 trong Bất đẳng thức và cực trị

$Đặt a=\frac{x}{y},b=\frac{y}{z},c=\frac{z}{x}.BĐT\Leftrightarrow \sum \frac{2xy}{x^2+3y^2}\leq \frac{3}{2}\Leftrightarrow \sum \frac{(x-y)^2+2y^2}{x^2+3y^2}\geq \frac{3}{2}.Lại có:\sum \frac{(x-y)^2}{x^2+3y^2}\geq \frac{(x-z)^2}{x^2+y^2+z^2};\sum \frac{2y^2}{x^2+3y^2}\geq \frac{(x+y+z)^2}{2(x^2+y^2+z^2)}.Ta Cm:\frac{(x-z)^2}{x^2+y^2+z^2}+\frac{(x+y+z)^2}{2(x^2+y^2+z^2)}\geq \frac{3}{2}\Leftrightarrow (x-y)(y-z)\geq 0 (1). Lập luân tương tự ta cần cm:(y-z)(z-x)\geq 0;(z-x)(x-y)\geq 0.Mà (x-y)(y-z)(y-z)(z-x)(z-x)(x-y)\geq 0 nên 1 trong 3 bđt nhỏ phải dúng=>dpcm$




#533912 $P=\sum \frac{a}{a^2+3}$

Đã gửi bởi DangHuyNgheAn on 20-11-2014 - 15:50 trong Bất đẳng thức và cực trị

Mình có cách làm đơn gian hon nhieu.




#533908 $P=\sum \frac{a}{a^2+3}$

Đã gửi bởi DangHuyNgheAn on 20-11-2014 - 15:43 trong Bất đẳng thức và cực trị

$Cho:a,b,c>0 thoaman:abc=1.Tim Max P=\sum \frac{a}{a^2+3}$




#533763 giai pt $5x^{2}-\left ( 3x+1 \right )\sqrt...

Đã gửi bởi DangHuyNgheAn on 19-11-2014 - 12:17 trong Phương trình, hệ phương trình và bất phương trình

Cai nay dat an phu la duoc ma.




#533369 $P=\sum \frac{1}{a^2-bc+1}$

Đã gửi bởi DangHuyNgheAn on 15-11-2014 - 22:25 trong Bất đẳng thức và cực trị

Cho $a,b,c>0$ thoả mãn:$ab+bc+ca=\frac{1}{3}$.Tim Max $P=\sum \frac{1}{a^2-bc+1}$




#532861 Đề Thi chọn đội tuyển Olympic toán 10 trường THPT BỈm sơn - Thanh Hóa

Đã gửi bởi DangHuyNgheAn on 11-11-2014 - 22:26 trong Tài liệu tham khảo khác

 ghê vây!




#532040 CMR:

Đã gửi bởi DangHuyNgheAn on 05-11-2014 - 21:56 trong Số học

$Cho (a-b,ab+1)=(a+b,ab-1)=1,a>b,a,b\epsilon N*. CMR: (a-b)^2+(ab+1)^2 khong chinhphuong. Ps:minhlamduocbainaymakhongungdk(a-b,ab+1)=1.Moinguoicoicoduockhong?$